Solving ODE by Laplace Transform: Where Did I Go Wrong?

Click For Summary
The discussion revolves around solving the ordinary differential equation (ODE) xy'' + y' + 4xy = 0 using the Laplace transform. The user attempts to derive the solution but encounters issues with negative signs in their calculations, particularly when differentiating terms involving L(y). They realize that correcting the derivative term leads to a more accurate formulation, allowing them to progress towards the correct solution. The conversation emphasizes the importance of careful differentiation in the application of the Laplace transform to ODEs. Ultimately, the user acknowledges their earlier mistake and seeks to continue the derivation correctly.
roughwinds
Messages
11
Reaction score
0
Member warned about posting without the homework template

Homework Statement


Use Laplace transform to solve the following ODE

Homework Equations


xy'' + y' + 4xy = 0, y(0) = 3, y'(0) = 0

The Attempt at a Solution


L(xy'') = -\frac{dL(y'')}{ds}

L(4xy) = -\frac{4dL(y)}{ds}

L(y'') = s²L(y) - sy(0) - y'(0) = s²L(y) -3s

L(y') = sL(y) - sy(0) - y(0) = sL(y) - 3

-\frac{d(s²L(y)-3s)}{ds} + sL(y)-\frac{4dL(y)}{ds} =0

-\frac{d(s²L(y))}{ds} + 3 + sL(y) - 3 -\frac{4dL(y)}{ds} =0

-\frac{d(s²L(y))}{ds} + sL(y) -\frac{4dL(y)}{ds} =0

-\frac{dL(y)(s²+4)}{ds} + sL(y) =0 (1)

\frac{dL(y)}{L(y)} = \frac{sds}{s²+4}

Integrating both sides

ln(L(y)) = \frac{ln(s²+4)}{2} + c

L(y) = c\sqrt{s²+4}

which won't lead me to the right answer.

I realized that if at (1) I use \frac{L(y)(s² + 4)}{ds} + sL(y) =0 instead I'll reach the right answer according to wolfram, but I can't figure out what I'm doing wrong to end up with that negative sign.

http://www.wolframalpha.com/input/?i=xy''+++y'+++4xy+=+0,+y(0)+=+3,+y'(0)+=+0
 
Last edited:
Physics news on Phys.org
roughwinds said:
xy'' + y' + 4xy = 0, y(0) = 3, y'(0) = 0

L(xy'') = -\frac{dL(y'')}{ds}

L(4xy) = -\frac{4dL(y)}{ds}

L(y'') = s²L(y) - sy(0) - y'(0) = s²L(y) -3s

L(y') = sL(y) - sy(0) - y(0) = sL(y) - 3

-\frac{d(s²L(y)-3s)}{ds} + sL(y)-\frac{4dL(y)}{ds} =0

-\frac{d(s²L(y))}{ds} + 3 + sL(y) - 3 -\frac{4dL(y)}{ds} =0

-\frac{s²L(y)}{ds} + sL(y) -\frac{4dL(y)}{ds} =0

-\frac{L(y)(s²+4)}{ds} + sL(y) =0 (1)

\frac{dL(y)}{L(y)} = \frac{sds}{s²+4}

Integrating both sides

ln(L(y)) = \frac{ln(s²+4)}{2} + c

L(y) = c\sqrt{s²+4}

which won't lead me to the right answer.

I realized that if at (1) I use \frac{L(y)(s² + 4)}{ds} + sL(y) =0 instead I'll reach the right answer according to wolfram, but I can't figure out what I'm doing wrong to end up with that negative sign.

http://www.wolframalpha.com/input/?i=xy''+++y'+++4xy+=+0,+y(0)+=+3,+y'(0)+=+0
Something went wrong from:
-\frac{d(s²L(y))}{ds} + 3 + sL(y) - 3 -\frac{4dL(y)}{ds} =0
to
-\frac{s²L(y)}{ds} + sL(y) -\frac{4dL(y)}{ds} =0

Check what you did with the ##-\frac{d(s²L(y))}{ds}## term.
 
  • Like
Likes roughwinds
Samy_A said:
Something went wrong from:
-\frac{d(s²L(y))}{ds} + 3 + sL(y) - 3 -\frac{4dL(y)}{ds} =0
to
-\frac{s²L(y)}{ds} + sL(y) -\frac{4dL(y)}{ds} =0

Check what you did with the ##-\frac{d(s²L(y))}{ds}## term.
It's supposed to be
-\frac{d(s²L(y))}{ds} + sL(y) -\frac{4dL(y)}{ds} =0
fixed it on the original post.
 
roughwinds said:
It's supposed to be
-\frac{d(s²L(y))}{ds} + sL(y) -\frac{4dL(y)}{ds} =0
fixed it on the original post.
Ok, so now continue your derivation from that.
 
  • Like
Likes roughwinds
Samy_A said:
Ok, so now continue your derivation from that.
-\frac{d(s²L(y))}{ds} = -2sL(y) - \frac{s²d(L(y))}{ds}
-2sL(y) - \frac{s²d(L(y))}{ds} + sL(y) -\frac{4dL(y)}{ds} =0
- \frac{s²d(L(y))}{ds} - sL(y) -\frac{4dL(y)}{ds} =0
\frac{s²d(L(y))}{ds} + sL(y) +\frac{4dL(y)}{ds} =0
\frac{d(L(y))(s²+4)}{ds} + sL(y) =0

Thanks, I initially solved as if s² wasn't part of the derivative.
 
Question: A clock's minute hand has length 4 and its hour hand has length 3. What is the distance between the tips at the moment when it is increasing most rapidly?(Putnam Exam Question) Answer: Making assumption that both the hands moves at constant angular velocities, the answer is ## \sqrt{7} .## But don't you think this assumption is somewhat doubtful and wrong?

Similar threads

  • · Replies 7 ·
Replies
7
Views
2K
  • · Replies 4 ·
Replies
4
Views
3K
  • · Replies 8 ·
Replies
8
Views
2K
  • · Replies 1 ·
Replies
1
Views
1K
  • · Replies 4 ·
Replies
4
Views
2K
Replies
9
Views
2K
  • · Replies 3 ·
Replies
3
Views
2K
  • · Replies 2 ·
Replies
2
Views
2K
  • · Replies 3 ·
Replies
3
Views
1K
Replies
9
Views
2K